Physics/Theory of Relativity questions

Started by AndyZ, May 16, 2012, 01:52:19 AM

Previous topic - Next topic

0 Members and 1 Guest are viewing this topic.

AndyZ

Looking for someone who's, if not well knowledgeable on the theory of relativity and experiments therein, at least knows a decent amount about physics and can answer a few questions which have been bugging me.  Really, I'll take what I can get if people can give evidence, reasoning, etc.

I'll pop down one for now, though I'm sure I'll have more:

If a grenade's shock wave normally travels at X miles an hour, and I throw it out of a plane travelling at 600mph, will the shock wave travel at (600+X-Air Resistance) miles an hour, or just X miles an hour?
It's all good, and it's all in fun.  Now get in the pit and try to love someone.

Ons/Offs   -  My schedule and A/As   -    My Avatars

If I've owed you a post for at least a week, poke me.

Oniya

I'm not much up on the physics of explosions (which involve a chemical reaction instead of merely a moving body), but I may be able to help with some of the other questions.  Did a lot of physics reading not too long ago (including some on relativity), and I still have my old physics textbook for the Newtonian stuff.
"Language was invented for one reason, boys - to woo women.~*~*~Don't think it's all been done before
And in that endeavor, laziness will not do." ~*~*~*~*~*~*~*~*~*~*~Don't think we're never gonna win this war
Robin Williams-Dead Poets Society ~*~*~*~*~*~*~*~*~*~*~*~*~*~Don't think your world's gonna fall apart
I do have a cause, though.  It's obscenity.  I'm for it.  - Tom Lehrer~*~All you need is your beautiful heart
O/O's Updated 5/11/21 - A/A's - Current Status! - Writing a novel - all draws for Fool of Fire up!
Requests updated March 17

gaggedLouise

#2
I'm no physicist but (like Oniya) I've read some about this over the years; also for what little worth it has, I know some people who have been doing research in astrophysics and have discussed this stuff with them in a relaxed way.

First off, 600 mph is nowhere near the kind of speed where you would begin to have relativistic changes of speed from different angles; 600 miles per second wouldn't be of any consequence either. And as long as the grenade is in the atmosphere, or building up to escape velocity (about 7miles/second from earth - let's suppose the planet in the example has this EV) it doesn't pick up the planet's orbit speed as any part of its own speed from the point of view of someone observing it from the planet. If it were observed from another planet much further outside, yes, then it would be seen to travel with that speed plus its own speed when it was orbiting "forwards" and then v=planetary orbit speed minus object's own moment speed, when it sort of went "around the bend" into the other half of its orbit.)

If it broke free completely from the gravity of the planet it had been sent from, and went into a path taking it away from it - like a probe for Mars or the outer planets - it would keep the speed and orbital direction it had at the point when it completely broke off the "home planet gravity field" which is *not* the same as the point where it enters into orbit around that planet: any satellite has to reach escape velocity to enter into orbit, but if it's lost in space later, near earth, or between the earth and the moon, it will still slip into orbit around the earth for many years. So it's only when they have gone some way beyond the distance to the moon (in earth's case) that these probes or whatever it is escape any substantial bending power from the earth on their paths through space.

Once it has got that far, and starts travelling freely through interplanetary space, if it has some engines of its own turned on during the entire trip to let's say the next planet, it won't pick up the home planet's orbit speed in any simple way, no matter from where it is being observed. A shock wave erupting from a point hat far out would be, I think, the "shockwave speed" added to the speed of the grenade just before it exploded, and theoretically it would travel back to its home planet, or on to another planet with that speed, if those planets were moving at a slower speed or moving towards the spreading circle of the shock wave, meeting it (actually I think any sizable planet in the inner parts of a planetary system, even Saturn in our own neighbourhood, would be moving much faster through its orbit than most probes or grenades could hope to travel - so they would often outrun the shockwave unless it happened "in front of" said planet). .

The trouble is, most probes we send out don't run on their own engines all the way out. The engines are shut off as soon as the ship or probe has been circling around earth a bit and then pushed itself off into an orbit aimed at the goal it is set to reach in a year or two. Loading Pioneer, Mariner or Cassini with fuel for travelling on their own power all the way through the solar system would be prohibitively expensive, even if it was nuclear fuel. With chemical rocket fuel it would be out of the question (the jury is still out on whether it could be practical to power the first manned trips to Mars with any kind of standard rocket fuel for the full length of the trip, though it owuld make the running time considerably briefer because you could just go the shortest way, pretty much a straight line). Once they have got up to a good level they just coast on the orbits they will get into through the gravity of the sun and the planets nearby, and the speed in that orbit is not a simple product of neither the home planet's orbital speed, nor the vessel's own maximum speed as it took off from the planet.

Good girl but bad  -- Proud sister of the amazing, blackberry-sweet Violet Girl

Sometimes bound and cuntrolled, sometimes free and easy 

"I'm a pretty good cook, I'm sitting on my groceries.
Come up to my kitchen, I'll show you my best recipes"

ExisD

#3
I'm someone who's studied engineering so I have a pretty hefty physics background, but it is mostly in physical systems. Though I don't specialize in explosives we did cover them briefly in a course. Within my experience we cared more about the total energy output from the explosion than the speed of the shockwave. Because of this I'll base my analysis off of one of the fragments sent flying and use its velocity as a basis for the velocity of the explosion. Note: I have no idea of what the exact correlation between the two is.



For the grenade question, the big questions here are which frame of reference are you looking from along with the base assumptions for the problem.

The base assumptions I'll use are as such: the explosion is perfectly spherical in nature as are the fragments, the planes is moving at exactly 600mph with no acceleration, no rotational velocity, and no rotational acceleration. All of these are assuming a fragment flies outward parallel to the x axis. If you were looking at one of other axis then X would have to be divided into multiple numbers based on the actual direction.

In all examples X will be speed of the fragment, Y will be the velocity loss from air friction, and Z will be the effects of gravity. In each of these examples the values of Y and Z are going to be based on time. As time increases the value these these will increase with Y having a maximum value of X(in the first two frames and X+600 in the last one) and Z having a maximum value of terminal velocity.

If you are considering the velocity of the explosion from a frame attached to the grenade with the x axis being in the same direction as the velocity of the plane and the z axis being perpendicular to the ground. Then the velocity of the fragment will be X-Y in the x axis and 0 in the z axis.

If you are considering the velocity of the explosion from a frame attached to the plane with the x axis being in the same direction as the velocity of the plane and the z axis being perpendicular to the ground. Then the velocity of the fragment will be X-Y in the x axis and Z in the z axis.

If you are considering the velocity of the explosion from a frame attached to the Earth with the x axis being in the same direction as the velocity of the plane and the z axis being perpendicular to the ground. Then the velocity of the fragment will be 600mph+X-Y in the x axis and Z in the z axis.

This is because when looking at velocity you ignore the velocity and acceleration of the object your frame of reference rests on. As an example when driving it doesn't look like you are moving, rather that the entire world besides you is moving. Yet when you are standing on the side of the road you appear stationary to yourself and all of the cars are moving.

Does that answer your question in a manner that makes sense?

Reason for edit: Posting while brain dead made me forget to name the frame in one of the maximums I listed.

As for why the reference frames are important to the theory of relativity. The frame you are looking at an objects velocity relative to its own is an important part of the problem to be solved.

Vekseid

Quote from: AndyZ on May 16, 2012, 01:52:19 AM
If a grenade's shock wave normally travels at X miles an hour, and I throw it out of a plane travelling at 600mph, will the shock wave travel at (600+X-Air Resistance) miles an hour, or just X miles an hour?

This isn't a relativity question.

When an explosion occurs, a small amount of solid matter turns into a large amount of gaseous matter at very high speed, displacing the surrounding air at the same speed. Here you determine 'X' to be the volume taken up in an amount of time given by the explosive's own velocity, which is dependent on the explosive, and of course, it's a volume, rather than something 'simple' like calculating the muzzle velocity of a rifle round. But this wave uses the grenade's frame of reference, and is 600+X.

Equations for this sort of thing with nuclear weapons are publicly available, but above-ground nuclear explosions have a lot to do with lighting the surrounding air on fire, making something a bit more slower than the mach ~4 missile (or whatever) the main reactant in the explosion.

AndyZ

Thanks for responding, folks ^_^

First off, I wanted to clarify that I realize there's a difference between general and special theories of relativity.  This part was more general.

If you're on a bike going 120mph, and you throw a baseball 90mph, then from the perspective of someone watching along the side of the road, the baseball will appear to travel 210mph.  Mostly I was wondering if that works the same for non-kinetic forces.  Then again, maybe an explosion is kinetic?

Sadly, I'm not educated enough to really understand all the formulas.  When I read through Einstein's book, I kinda glazed over the letters and numbers; the principles made sense, the calculations not so much.
It's all good, and it's all in fun.  Now get in the pit and try to love someone.

Ons/Offs   -  My schedule and A/As   -    My Avatars

If I've owed you a post for at least a week, poke me.

gaggedLouise

#6
Are you thinking of explosions that would happen in outer space (between planets) or in the planet's atmosphere? The pattern and speed of spreading would be quite different, both because there is no continuous medium in outer space (such as air, water or earth) to forward the explosion, and because the changes in speed of the object before it explodes would be very affected by its being up close to a planet. Comets and meteorites that approach earth increase their speed and temperature as they get closer to us, because they are pulled in by the gravity force of earth. They gain more momentum - and then it might end up in their being repelled and tossed out into space again, so they don't crash down on us.

Good girl but bad  -- Proud sister of the amazing, blackberry-sweet Violet Girl

Sometimes bound and cuntrolled, sometimes free and easy 

"I'm a pretty good cook, I'm sitting on my groceries.
Come up to my kitchen, I'll show you my best recipes"

Darius

Ok, assuming something the size of a round concussion grenade to alleviate all kinds of drag problems.

If you throw the grenade out perpendicular to the plane the air pressure against the mass of the grenade will slow it sufficiently in the 4-8 seconds (depending on fusing) prior to the explosion to make the velocity of the grenade insignificant compared to the speed of the shock wave it generates.

If you throw it out the back of your plane at that speed you likely get the same results.

If you throw it forward from the plane you asking to be hit in the face with a hand grenade, or to have it sucked through the air intake of your jet engine with sufficiently poor results that you will likely not care about the speed of shock waves.
When the avalanche has started, the pebbles no longer get to vote.
Ons and Offs
absence to make hearts grow fonder
Story ideas
Darius & His Ramblings
No one is an unjust villain in their own mind… we are all the hero of our own story.” A Lucio

Samnell

Quote from: AndyZ on May 18, 2012, 09:54:31 PM
If you're on a bike going 120mph, and you throw a baseball 90mph, then from the perspective of someone watching along the side of the road, the baseball will appear to travel 210mph.  Mostly I was wondering if that works the same for non-kinetic forces.  Then again, maybe an explosion is kinetic?

Explosions involve kinetic energy. If there's motion, it's kinetic.

AndyZ

Quote from: gaggedLouise on May 18, 2012, 10:50:01 PM
Are you thinking of explosions that would happen in outer space (between planets) or in the planet's atmosphere? The pattern and speed of spreading would be quite different, both because there is no continuous medium in outer space (such as air, water or earth) to forward the explosion, and because the changes in speed of the object before it explodes would be very affected by its being up close to a planet. Comets and meteorites that approach earth increase their speed and temperature as they get closer to us, because they are pulled in by the gravity force of earth. They gain more momentum - and then it might end up in their being repelled and tossed out into space again, so they don't crash down on us.

I understand about comets speeding up as gravity suddenly kicks in, especially before they have any wind resistance which would cause terminal velocity, but I don't really understand how explosions in space are different from explosions on Earth.  If I must pick one, though, I'd go with explosions on Earth, though I'd like to know how each are different.

Quote from: Darius on May 18, 2012, 10:56:30 PM
Ok, assuming something the size of a round concussion grenade to alleviate all kinds of drag problems.

If you throw the grenade out perpendicular to the plane the air pressure against the mass of the grenade will slow it sufficiently in the 4-8 seconds (depending on fusing) prior to the explosion to make the velocity of the grenade insignificant compared to the speed of the shock wave it generates.

If you throw it out the back of your plane at that speed you likely get the same results.

If you throw it forward from the plane you asking to be hit in the face with a hand grenade, or to have it sucked through the air intake of your jet engine with sufficiently poor results that you will likely not care about the speed of shock waves.

See, I'm cursed with the curiosity inherent on problems which may not be practical.  For example, I want to know what happens after I die, even though I'm not willing to kill myself purely to find out.  Thanks, though.

Quote from: Samnell on May 18, 2012, 11:08:45 PM
Explosions involve kinetic energy. If there's motion, it's kinetic.

Does that apply to anything that moves, including light, particles and things like that?
It's all good, and it's all in fun.  Now get in the pit and try to love someone.

Ons/Offs   -  My schedule and A/As   -    My Avatars

If I've owed you a post for at least a week, poke me.

Darius

Quote from: AndyZ on May 18, 2012, 11:36:46 PM
Does that apply to anything that moves, including light, particles and things like that?

Absolutely
When the avalanche has started, the pebbles no longer get to vote.
Ons and Offs
absence to make hearts grow fonder
Story ideas
Darius & His Ramblings
No one is an unjust villain in their own mind… we are all the hero of our own story.” A Lucio

Samnell

Quote from: AndyZ on May 18, 2012, 11:36:46 PM
See, I'm cursed with the curiosity inherent on problems which may not be practical.  For example, I want to know what happens after I die, even though I'm not willing to kill myself purely to find out.  Thanks, though.

That's a biology question. :) Granted biology exists inside physics, just like chemistry, but it's running at different levels of complexity.

Quote from: AndyZ on May 18, 2012, 11:36:46 PM
Does that apply to anything that moves, including light, particles and things like that?

Like Darius said, yes. If it helps, kinetic energy is a type of energy that includes lots of other things that we call energy: heat, radiation, electricity, etc. All those guys are in motion. With regard to kinetic energy, its opposite number is potential energy. When objects are in motion we're seeing their potential energy turn into kinetic energy and kinetic into potential, minus some energy that will leave the system from things like friction.

AndyZ

I guess the issue I've had is stuff like that I've heard that the speed of sound is constant through air.  Is this incorrect as well?  Does a moving person's voice move faster than a still person's voice?
It's all good, and it's all in fun.  Now get in the pit and try to love someone.

Ons/Offs   -  My schedule and A/As   -    My Avatars

If I've owed you a post for at least a week, poke me.

Vekseid

Quote from: AndyZ on May 18, 2012, 09:54:31 PM
Thanks for responding, folks ^_^

First off, I wanted to clarify that I realize there's a difference between general and special theories of relativity.  This part was more general.

None of what you are asking involves general relativity in any sense. The only part of relativity you need to know for any of this is the concept of frames of reference. You measure things in miles per hour and there are less than seven digits involved, relativity doesn't really cause any issues except for things like GPS.

Quote from: AndyZ on May 19, 2012, 11:20:49 PM
I guess the issue I've had is stuff like that I've heard that the speed of sound is constant through air.  Is this incorrect as well?  Does a moving person's voice move faster than a still person's voice?

Air can move. So does spacetime, of course, but these are exceptional situations (spacetime expands at some 70 km/sec/megaparsec, spacetime gets 'swallowed' by a black hole).

So your question about the grenade's shock wave - eventually, yes, it's leading edge won't be any faster than the speed of sound. But how fast this is in which direction depends on its altitude (the current pressure), which way the wind is blowing and how fast, etc.

Thinking of the speed of sound as being a microcosm of the speed of light in spacetime is useful for describing certain concepts - the Doppler effect, for example. But there's no observed and verifiable physical transformation of the same sort that occurs as things approach the speed of light.

Samnell

Quote from: AndyZ on May 19, 2012, 11:20:49 PM
I guess the issue I've had is stuff like that I've heard that the speed of sound is constant through air.  Is this incorrect as well?  Does a moving person's voice move faster than a still person's voice?

I don't have much to add to what's been said already, but it might be helpful to remember that sound is just vibration going through a medium. There's no sound particle generated by noise. It's just like tossing a stone in still water and watching the waves emanate from the impact site.

Light is different. There is actual light stuff which behaves in some ways that are hard to understand without knowing the math. Which I do not. :)

Oniya

Light has instances where it behaves like a wave (interference patterns, red-shifts) and instances where it behaves like a particle (photons can be detected individually) - this is one of the reasons that talking about things involving light get messy.
"Language was invented for one reason, boys - to woo women.~*~*~Don't think it's all been done before
And in that endeavor, laziness will not do." ~*~*~*~*~*~*~*~*~*~*~Don't think we're never gonna win this war
Robin Williams-Dead Poets Society ~*~*~*~*~*~*~*~*~*~*~*~*~*~Don't think your world's gonna fall apart
I do have a cause, though.  It's obscenity.  I'm for it.  - Tom Lehrer~*~All you need is your beautiful heart
O/O's Updated 5/11/21 - A/A's - Current Status! - Writing a novel - all draws for Fool of Fire up!
Requests updated March 17

gaggedLouise

#16
Quote from: AndyZ on May 18, 2012, 11:36:46 PM
I understand about comets speeding up as gravity suddenly kicks in, especially before they have any wind resistance which would cause terminal velocity, but I don't really understand how explosions in space are different from explosions on Earth.  If I must pick one, though, I'd go with explosions on Earth, though I'd like to know how each are different.


I guess I was thinking of how the thrust of the explosion moves forward. The "wave" part of a powerful explosion (or an eruption, for that matter) will reach much further than the direct throw-out of splinters of the object that exploded, provided that there is a medium the wave can move in. On earth we don't think of that because there's always a continuing medium of some kind: water, air, earth, so the wave will have something to move in - you have to offset the resistance of that medium of course.

In space there is no physical medium, so a wave of matter, or even of sound, can't really travel very far unless it has a carrier that keeps on pushing it when the opening impact has gone down (it does keep travelling until it hits into something that stops it or bounces it, but it's no longer traveling "on its own speed", only on a gravity controlled orbit). An explosion in outer space wouldn't make any sound - I reckon you knew that already - since there is no atmosphere or liquid for the sound waves to travel in. And with solid matter - parts of a bomb or a blown-up spaceship - the concerted force of an explosion (if it were like any man-made explosion we can set off today) would soon be gone: every individual piece of scrap would just slip off into a trajectory that would really be controlled by the gravity of distant objects, such as planets and the star of the system. I don't think you would get any "added speed" to the explosion due to the orbit and speed the object was on at the moment before it exploded, as would have happened on earth.

With cases like the remains of a supernova, where there's sheets of gas that keep moving at high speed in a clearly visible way for many thousands of years after the star died, the force that "carries" the motion is heated gas and the ability to keep up chemical reactions and ionization long after the explosion, and it creates a wave motion in the interstellar medium. So it's not just a mechanical thrust from the point where the star stood before it exploded, it's mostly a wave of heat and radiation - unlike matter and sound, these do not need a medium to keep actively moving forward.

Good girl but bad  -- Proud sister of the amazing, blackberry-sweet Violet Girl

Sometimes bound and cuntrolled, sometimes free and easy 

"I'm a pretty good cook, I'm sitting on my groceries.
Come up to my kitchen, I'll show you my best recipes"

Oniya

Well - radiation doesn't need a medium, since it provides its own (as alpha and beta particles).  Heat, which is created by kinetic energy, is stopped by a vacuum.  This is actually why a Thermos bottle works - a thin layer of vacuum between the inner and outer walls prevents the kinetic energy from your hot coffee from getting out as quickly (some is still transferred through the points of contact between the inner shell and the outer shell, but that's limited by design.)
"Language was invented for one reason, boys - to woo women.~*~*~Don't think it's all been done before
And in that endeavor, laziness will not do." ~*~*~*~*~*~*~*~*~*~*~Don't think we're never gonna win this war
Robin Williams-Dead Poets Society ~*~*~*~*~*~*~*~*~*~*~*~*~*~Don't think your world's gonna fall apart
I do have a cause, though.  It's obscenity.  I'm for it.  - Tom Lehrer~*~All you need is your beautiful heart
O/O's Updated 5/11/21 - A/A's - Current Status! - Writing a novel - all draws for Fool of Fire up!
Requests updated March 17

gaggedLouise

#18
Okay, thanks for enlightening  me on that one, Oniya, but with "absence of a medium" (having a void, that is) I wasn't thinking of a perfect vacuum, which is something that really has to be constructed by some engineers or physicists - it takes considerable skill to create a reliable lab vacuum that CERN would recognize - but rather discontinuity and sparseness of medium, a region of space where the particles do not really add up to a continuing medium that the waves in question could travel in. If you only have a hundred atoms per litre/cubic decimeter of space, excepting the sudden burst of foreign particles from the exploding object, then it stops any sound wave and probably doesn't do much in order to carry a grenade tremor forward

Good girl but bad  -- Proud sister of the amazing, blackberry-sweet Violet Girl

Sometimes bound and cuntrolled, sometimes free and easy 

"I'm a pretty good cook, I'm sitting on my groceries.
Come up to my kitchen, I'll show you my best recipes"

Oniya

True enough (and the void in the Thermos bottle probably isn't CERN-level vacuum either), but in order for heat to be transferred, the particles need to collide with each other often enough to transfer kinetic energy.  Heat from a non-nuclear explosion outside an atmosphere is going to drop off incredibly fast.  Now, radiation (being particulate in itself, and generating new particles as atoms are broken up by the collisions) can transfer/generate heat, but you're still dealing with a rapid increase in volume that those particles are bouncing around in.
"Language was invented for one reason, boys - to woo women.~*~*~Don't think it's all been done before
And in that endeavor, laziness will not do." ~*~*~*~*~*~*~*~*~*~*~Don't think we're never gonna win this war
Robin Williams-Dead Poets Society ~*~*~*~*~*~*~*~*~*~*~*~*~*~Don't think your world's gonna fall apart
I do have a cause, though.  It's obscenity.  I'm for it.  - Tom Lehrer~*~All you need is your beautiful heart
O/O's Updated 5/11/21 - A/A's - Current Status! - Writing a novel - all draws for Fool of Fire up!
Requests updated March 17

gaggedLouise

#20
Spirits in Space

Speaking of sparse interstellar medium that yet adds up to something, I remember reading somewhere that there are enough ethanol molecules in the interstellar space of the Milky Way alone that they would make up an estimated billion of litres of erm, liquid (raw) liquor if they were pulled together.  :-) The trouble being, of course, to find them.

Good girl but bad  -- Proud sister of the amazing, blackberry-sweet Violet Girl

Sometimes bound and cuntrolled, sometimes free and easy 

"I'm a pretty good cook, I'm sitting on my groceries.
Come up to my kitchen, I'll show you my best recipes"

AndyZ

Just wanted to say thanks to people who are posting.  Haven't had much to say but I'm reading it all and trying my best to understand.
It's all good, and it's all in fun.  Now get in the pit and try to love someone.

Ons/Offs   -  My schedule and A/As   -    My Avatars

If I've owed you a post for at least a week, poke me.

Oniya

If you want some offline references, I recommend the 'Blackboard Book' E=Mc2, and The Dancing Wu Li Masters.  The first one is a book designed as a refresher of basic concepts - not much math, a few tables as I recall, and a good dose of humor.  The second one is specifically geared as physics without the math - almost as philosophy.
"Language was invented for one reason, boys - to woo women.~*~*~Don't think it's all been done before
And in that endeavor, laziness will not do." ~*~*~*~*~*~*~*~*~*~*~Don't think we're never gonna win this war
Robin Williams-Dead Poets Society ~*~*~*~*~*~*~*~*~*~*~*~*~*~Don't think your world's gonna fall apart
I do have a cause, though.  It's obscenity.  I'm for it.  - Tom Lehrer~*~All you need is your beautiful heart
O/O's Updated 5/11/21 - A/A's - Current Status! - Writing a novel - all draws for Fool of Fire up!
Requests updated March 17

Exelion

Well, if you start to REALLY think about it...that grenade and helicopter are on the earth, which is currently travelling at something like 66,000 mph..in a galaxy traveling tens of thousands of mph...

I try not to think about these things. My head might explode.

dutchprof

Quote from: gaggedLouise on May 24, 2012, 03:50:19 AM
In space there is no physical medium, so a wave of matter, or even of sound, can't really travel very far unless it has a carrier that keeps on pushing it when the opening impact has gone down (it does keep travelling until it hits into something that stops it or bounces it, but it's no longer traveling "on its own speed", only on a gravity controlled orbit). An explosion in outer space wouldn't make any sound - I reckon you knew that already - since there is no atmosphere or liquid for the sound waves to travel in. And with solid matter - parts of a bomb or a blown-up spaceship - the concerted force of an explosion (if it were like any man-made explosion we can set off today) would soon be gone: every individual piece of scrap would just slip off into a trajectory that would really be controlled by the gravity of distant objects, such as planets and the star of the system. I don't think you would get any "added speed" to the explosion due to the orbit and speed the object was on at the moment before it exploded, as would have happened on earth.
In otherwise empty space, matter particles resulting from an explosion might travel pretty far. In principle, they would travel indefinitely at a constant speed in a straight line; that is different than on earth, where interaction with surrounding air would slow debris down rather quickly.

In the gravitational field of a planet or star, the gravitational force would affect the trajectory of the debris. However, if you look at an explosion from a perspective that is in free fall, this falling motion would not be visible. (Compare to the motion of astronauts in the Space Station: they are in free fall and, contrary to popular belief, still experience about 98% of the earth's usual gravitational pull, but since the camera falls along with them they appear to float.) The only difference between an explosion in truly empty space and near a planet is due to "tidal forces", which seem to pull the exploding matter apart if it is spread out over larger distances. These tidal forces are due to the fact that the gravitational field is not constant but has varying strength and direction. These forces are usually rather weak.

If a satellite or spaceship would explode while in orbit around a planet, the debris particles would on average have the same speed and direction as the original object; this follows from the law of conservation of momentum. If you would travel in a second ship along with the exploding object, it would appear (from your perspective) that the explosion is symmetric, but an outsider would see the explosion move. In particular, when the debris impacts a nearby planet, they have more speed in the original direction of the exploding object; although air resistance is likely to reduce this forward speed before impact.

If a very large amount of matter explodes in otherwise empty space, the debris will gradually slow down because the particles attract each other. However, gravitational forces are very weak, so that this only strongly affects planetary and stellar quantities of matter. If debris moves outward at a speed faster than the escape speed at the surface of the object, it will be able to escape this gravitational pull and keep moving outward indefinitely; it will be slowed down a bit by gravity but will not fall back. For a 100-m radius, 1-million ton spaceship, this escape speed is about 0.1 mph.

AriDarrow

Ok, here's one that I've been curious about for a while. I get the whole space/time thing in that the faster something moves the slower time goes... and the whole idea of wormholes. My question comes into the spot right here... if someone made a wormhole pair, sent one out on a ship (wormhole X) at ~speed of light then return so that for the one that traveled (wormhole X), only a few hours passed while on Earth (wormhole Z) several years passed... so, in essence, we'd be able to travel between two times using those wormholes, right?

So, if that is true, then while X is in transit, and we were able to look through those wormholes and see the other side just fine, what does physics say we'd see from Earth? Does it say that we'd see the traveling side in real time to us or would it be slowed down to the point where it appears frozen? The way I am seeing it in my head, we'd have to see the other side in real time for the time travel to be possible.

Personally, I don't see time travel being possible. It makes the universe feel like a very messy place and all I've read and heard of physics at all levels, if something is messy, it is usually wrong. I know, couch-potato physics ideas like my own are usually missing some big peaces of the puzzle.

Now, for it to feel more... 'pretty?'... not right word, but at almost 5am, it will do... in my head, I can see the universe having time as a constant. Just hear me out a bit before pointing out that I am wrong and experiments have proven such. Time being a constant in that we can't escape the 'now'. Everything, be it billions of light years away or in the next room, all happens 'now'. However, that doesn't restrict us from perceiving or moving through now at a faster or slower rate.

Lets go back to those wormholes and see if I can't try and explain myself a little better by using examples.

As X moves near the speed of light, instead of it moving through time faster, it is actually still in the 'now', just moving really really slowly through 'now'. The same time dilation effects would still be seen and when returned to Earth, the same time differences would be seen, except that both wormholes would still be in the 'now'. There wouldn't be any time travel. While in transit, if we looked into Z to see what was happening on X, what we would see would be a very nearly frozen in time place as it traveled through 'now' at a very slow rate of speed even though it was traveling at a very high rate of speed through space.

I am just not sure if there are proven meth equations that show one way or the other, or even what kind of experiments could be had which would say one way or the other. Again, I've read a few books on this (they made my brain implode) and have watched many videos (Ted.com is awesome) but I've not found any that have really said anything that the way I'm seeing things is backwards. Again, in my head, it all works out well enough that it doesn't conflict with any theory that I know about.

Ideas anyone or did I just lose everyone with my random 5am ramblings... thanks ahead of time.

Vekseid

Quote from: AriDarrow on June 15, 2012, 05:00:05 AM
Ok, here's one that I've been curious about for a while. I get the whole space/time thing in that the faster something moves the slower time goes... and the whole idea of wormholes. My question comes into the spot right here... if someone made a wormhole pair, sent one out on a ship (wormhole X) at ~speed of light then return so that for the one that traveled (wormhole X), only a few hours passed while on Earth (wormhole Z) several years passed... so, in essence, we'd be able to travel between two times using those wormholes, right?

By just using Relativity, yes.

Quote
So, if that is true, then while X is in transit, and we were able to look through those wormholes and see the other side just fine, what does physics say we'd see from Earth? Does it say that we'd see the traveling side in real time to us or would it be slowed down to the point where it appears frozen? The way I am seeing it in my head, we'd have to see the other side in real time for the time travel to be possible.

Remove the wormhole from the equation for a moment - it isn't necessary.

Someone is moving at .86 of c away from you. This gives a nice time dilation factor of 2.

8 seconds passes for you, you see that 4 seconds has passed for them. Likewise, 8 seconds passes for them, they see that 4 seconds has passed for you. These are both perfectly accurate statements about the Universe, but for as long as you are moving apart at .86 of c, you will each see each other as experiencing time at half of the 'proper' rate.

This is where time travel rears its ugly head - both of your frames of reference are perfectly legitimate. So if you have some sort of ansible to communicate with, you send a message at t=8, the target gets it at t=4, and you get the reply at t=2...

Quote
Personally, I don't see time travel being possible. It makes the universe feel like a very messy place and all I've read and heard of physics at all levels, if something is messy, it is usually wrong. I know, couch-potato physics ideas like my own are usually missing some big peaces of the puzzle.

Meaningful FTL, Relativity, Causality, pick two. There are some very limited exceptions - you can use wormholes whose faces are fixed with respect to each other, for example (trying to do the typical ansible trick will cause the face of the wormhole to be deformed from your perspective, thus preserving causality), but otherwise, if meaningful FTL is possible in a universe that still maintains causality despite it, then something must be wrong with Relativity.

Note that what's wrong can certainly still permit our current understanding of Relativity - a preferred frame of reference that exists outside of our Universe is not observable, and thus, Relativity still holds for our 'limited' purview but there would be a point at which a new theory had to accommodate observations that occurred if/when some observable FTL phenomenon was discovered.

Note that there are faster-than-light phenomenon within our universe - matter falling into a black hole and the expansion of space - but these things are hidden from observers because they represent spacetime being deformed at such speeds.

Quote
Now, for it to feel more... 'pretty?'... not right word, but at almost 5am, it will do... in my head, I can see the universe having time as a constant. Just hear me out a bit before pointing out that I am wrong and experiments have proven such. Time being a constant in that we can't escape the 'now'. Everything, be it billions of light years away or in the next room, all happens 'now'. However, that doesn't restrict us from perceiving or moving through now at a faster or slower rate.

The first thing anyone should study when looking into Relativity is the concept of frames of reference. Your idea of 'now', no matter how constant you think it is, is still a part of that frame.

Quote
Lets go back to those wormholes and see if I can't try and explain myself a little better by using examples.

As X moves near the speed of light, instead of it moving through time faster, it is actually still in the 'now', just moving really really slowly through 'now'.

But Earth seems to be moving really slowly, from the perspective of the people on the other end of the wormhole. It isn't until they bring themselves back into Earth's frame (or at least close enough) that there can be any agreement. If Earth started zipping off after them at the same speed after several years, it would be the Earthlings who had very little time pass.

A 'true now' still represents a preferred frame - and is still verboten by Relativity. No concept of such that exists within our Universe is permissible by our observations.

AriDarrow

Quote from: Vekseid on June 15, 2012, 01:42:41 PM
By just using Relativity, yes.

Remove the wormhole from the equation for a moment - it isn't necessary.

Someone is moving at .86 of c away from you. This gives a nice time dilation factor of 2.

8 seconds passes for you, you see that 4 seconds has passed for them. Likewise, 8 seconds passes for them, they see that 4 seconds has passed for you. These are both perfectly accurate statements about the Universe, but for as long as you are moving apart at .86 of c, you will each see each other as experiencing time at half of the 'proper' rate.

This is where time travel rears its ugly head - both of your frames of reference are perfectly legitimate. So if you have some sort of ansible to communicate with, you send a message at t=8, the target gets it at t=4, and you get the reply at t=2...

Meaningful FTL, Relativity, Causality, pick two. There are some very limited exceptions - you can use wormholes whose faces are fixed with respect to each other, for example (trying to do the typical ansible trick will cause the face of the wormhole to be deformed from your perspective, thus preserving causality), but otherwise, if meaningful FTL is possible in a universe that still maintains causality despite it, then something must be wrong with Relativity.

Yeah, figured there was a lot I wasn't catching. Didn't think that wormholes fell into that 'FLT' category. I always thought of it as a ... non-travel? You enter one point, come out the other having gone a great distance but actually never did any traveling due to those two points being the same point, but at two different location.




Quote
But Earth seems to be moving really slowly, from the perspective of the people on the other end of the wormhole. It isn't until they bring themselves back into Earth's frame (or at least close enough) that there can be any agreement. If Earth started zipping off after them at the same speed after several years, it would be the Earthlings who had very little time pass.

This one is hard for me to wrap my mind around. So, being on the ship, traveling at near the speed of light, everyone looking out would see everything going real slow, the same as anyone looking into the ship? I can't understand how that would happen, because as they slowed down, there would be at some point where everything would have to either jump or move way faster than the time differences should be for it to all come back into proper perspective.

(I know these aren't the actual correct time differences, but using them as a rough example)

From Earth, they are gone 100 years, on the ship they are only gone a few days. From either perspective time is moving really slow? At least that's how you made it sound from that part I quoted of what you said.

AndyZ

Try wrapping your mind around it this way, AriDarrow:

No matter how fast they're going, no matter how much time dilates, light always appears to be travelling at c.

Whether the person is standing still or moving parallel to a laser beam, that laser beam appears to be travelling at the same speed.
It's all good, and it's all in fun.  Now get in the pit and try to love someone.

Ons/Offs   -  My schedule and A/As   -    My Avatars

If I've owed you a post for at least a week, poke me.

Vekseid

Quote from: AriDarrow on June 15, 2012, 04:15:00 PM
Yeah, figured there was a lot I wasn't catching. Didn't think that wormholes fell into that 'FLT' category. I always thought of it as a ... non-travel? You enter one point, come out the other having gone a great distance but actually never did any traveling due to those two points being the same point, but at two different location.

The definition of FTL, for practical purposes, is that information moves between two points faster than a photon can. So yes, wormholes are generally regarded as an FTL mechanism.

Quote
This one is hard for me to wrap my mind around. So, being on the ship, traveling at near the speed of light, everyone looking out would see everything going real slow, the same as anyone looking into the ship? I can't understand how that would happen, because as they slowed down, there would be at some point where everything would have to either jump or move way faster than the time differences should be for it to all come back into proper perspective.

Yes.

If it helps, picture, instead of Earth and a spaceship, just two spaceships in an endless void.

Quote
(I know these aren't the actual correct time differences, but using them as a rough example)

From Earth, they are gone 100 years, on the ship they are only gone a few days. From either perspective time is moving really slow? At least that's how you made it sound from that part I quoted of what you said.

They look back on Earth after those days, they've only seen a few moments pass on Earth.

And this is why talking about wormholes tends to cause headaches with causality. Whose frame of reference is the correct one? Relativity says both, but if you want to maintain causality, you need to say 'neither'. Earth's can't be valid, else we could play this trick out over gigaparsecs and end up with even more ludicrous results.

AriDarrow

Ah, I see where we are differing. If I am understanding you right, you are talking about looking at the Earth from the ship, that would cause the differences in what is seen with the flow of time. That's not what I am talking about though, my question lies in what is seen through the wormhole.

From what I understand of some views of a wormhole, they are in effect opposite sides of a 2 dimensional point in space, enter one side, come out the other, no space traveled even though the two sides could be light years apart.

If what would be seen from the ship looking back at Earth would be the same as what would be seen through the wormholes, then wouldn't that show time travel by such a method to be impossible and lean credence to the idea that time, or the 'now' that we are in, is 'static'? That's not a good way of phrasing it... but the time of night isn't letting my mind think of a better way.

The reason I ask that it not mattering if we view from the ship back to Earth or view through the wormhole, that what is seen would be the same is due to pretty much every scientist that I've seen talk about wormholes say it'd be otherwise due to that being the strongest possibility for time travel.

I do hope I was able to make my question a bit more clear on this and you see what I am talking about better.

Vekseid

Quote from: AriDarrow on June 16, 2012, 02:17:39 AM
Ah, I see where we are differing. If I am understanding you right, you are talking about looking at the Earth from the ship, that would cause the differences in what is seen with the flow of time. That's not what I am talking about though, my question lies in what is seen through the wormhole.

I am answering what is seen through the wormhole.

You fly away at .9999... of c (whatever level of Lorentz contraction you need for that), you see Earth through the wormhole as if the planet was nearly standing still, the light so redshifted you need to do hour-long exposures in pure darkness to know what's going on at the other end. A few days pass for you, you know you've moved 100 light-years from Earth's perspective... but Earth looks like mere seconds have passed. You bring yourself to a stop, and you and Earth both agree on the rate of time progression, but the Earth in the wormhole is still a century into the past from where someone on Earth looking into the now-non-redshifted wormhole is.


AriDarrow

I really am trying to wrap my mind around this, and I think I see what you are saying. Just trying to picture it in my head on how it works. Never been one for trying to use the lingo of a field of study to try and explain it to someone who isn't a student of that field.

I just hitting a wall as to seeing how it would actually make a difference to a wormhole. I can see everything else just fine, but to me that one thing just throws it for a loop. As I said, the idea of time travel just seems to make everything feel messy but the rest of physics, even quantum physics, is very eloquent and beautiful.

Being that it is a 2 dimensional surface, so there being no space between the two sides, it puzzles me why having one side traveling at near the speed of light or sitting right next to it, would make any difference to the flow of time that you could see from the other side. As you are saying, it's is relative, so two rockets, each traveling at half the speed of light in opposite directions from each other, you look from one to the other, you see the other as if it were traveling at the speed of light away.

So, you look through the wormhole, and see an image that isn't moving any faster in relation to what you are, but the other side is actually traveling at great speeds away. Why would what you see through the wormhole be the same kind of image that you would see from the ship looking back? That's the part I am not grasping. You mention red-shift, c, Lorentz contraction... etc... those words have no meaning to me due to, as I said, not having taken any official studies in the field. Is there a way that you can put it, describe a scene, a story maybe, but without any of the jargon? Maybe that could help me wrap my mind around why the image you see through a wormhole, which seems like it wouldn't be traveling any speed in relation to it's other half, wouldn't differ from what could be seen from the ship itself back to the planet.

AriDarrow

Well, it appears that I have managed to work my mind around the whole wormhole issue I was having. Don't really like the results that I can see though.

The ship relative to Earth, without the wormhole, time dilation takes effect, appears to slow down... etc. However, with the wormhole, in relation to the two sides, no traveling is taking place. Step through one side, you are on the other. What a person would see in looking in one side is real time on the other. So, time travel would be possible because the passage of time on the ship, say one year, would be what would be seen when the ship returns say 100 years later to Earth. So, in the past on Earth, a person would be looking 100 years into the future through the wormhole.

That is, if the actual results of said experiment would show the same results. Even today, experiments have shown that actual experiments of what was only theory give results different than what was expected, that is what I hope will result when this kind of a test is actually possible. As messy as I feel time travel make the universe, it may actually be a possible result.

Vekseid

The physical definition of travel is that information moves from point a to point b. Matter is a form of information.

If it's faster than a photon traveling the same route, it is, by definition, faster than light. Traveling is taking place, and your declaration that it isn't is physically nonsensical - information that was on Earth is now 100 light-years away, or vise-versa. That is travel. It violates the dominant energy condition by its very occurrence.




Your problem isn't that some sort of FTL travel is impossible while preserving causality. Your problem is in the assumption that Earth would be a preferred frame in such a scenario. This is the most common flaw of home-grown FTL constructs for science fiction. In order to have FTL and Causality, Relativity must, at some point, be wrong. Okay, this is fine - but it's certainly true for the Universe as we currently observe it, and, most critically, we observe no preferred frame, which is the simplest means by which FTL and causality can be permitted.

So that frame of reference, if it exists, cannot exist 'within' our universe. This means that your idea of a Universal 'now' can only, at best, be approximated (some researchers have posited that the IGM (Inter-galactic medium would best represent such a thing)... so can have your FTL and your Causality, but it means that certain things that people use in their descriptions of such things - like 'fifty times the speed of light' do not actually make sense. The speed of light remains a sort of infinite quantity, even in a universe where FTL and Causality still hold.

AriDarrow

Quote from: Vekseid on June 16, 2012, 02:29:41 PM
The physical definition of travel is that information moves from point a to point b. Matter is a form of information.

If it's faster than a photon traveling the same route, it is, by definition, faster than light. Traveling is taking place, and your declaration that it isn't is physically nonsensical - information that was on Earth is now 100 light-years away, or vise-versa. That is travel. It violates the dominant energy condition by its very occurrence.




Your problem isn't that some sort of FTL travel is impossible while preserving causality. Your problem is in the assumption that Earth would be a preferred frame in such a scenario. This is the most common flaw of home-grown FTL constructs for science fiction. In order to have FTL and Causality, Relativity must, at some point, be wrong. Okay, this is fine - but it's certainly true for the Universe as we currently observe it, and, most critically, we observe no preferred frame, which is the simplest means by which FTL and causality can be permitted.

So that frame of reference, if it exists, cannot exist 'within' our universe. This means that your idea of a Universal 'now' can only, at best, be approximated (some researchers have posited that the IGM (Inter-galactic medium would best represent such a thing)... so can have your FTL and your Causality, but it means that certain things that people use in their descriptions of such things - like 'fifty times the speed of light' do not actually make sense. The speed of light remains a sort of infinite quantity, even in a universe where FTL and Causality still hold.


Not sure I understood everything you were talking about there, again, using the terminology that I am not fully educated in is only adding to the confusion. The only reason I am using Earth instead of 2 ships or even 2 planets, is just one for convince. We live on Earth, and for the most part, it is the center of all that happens to us. Familiar starting point and something we can all feel safe using.

I understand that anything going over a distance is travel, but what distance is traveled between the wormhole sides? It's a 2 dimensional space, right? Someone walking at say, 2mph going into one side is walking the same 2mph coming out the other and if those ends of the wormhole are at opposite sides of the universe, does that mean that they have said ungodly high speed of some FTL travel, or were they only going at 2mph at all points?

It's well known that there are galaxies out there that are traveling away from us at speeds greater than the speed of light. It's not due to said galaxy actually traveling faster than the speed of light, but due to our mutual speeds traveling away from a random set neutral point between our two galaxies and the expansion of the universe. At no point is any going FTL, but combine all the factors and FTL is observed. I don't like saying it this way, but it's one of the better ways of putting, all speed is relative. Two ships traveling in the same direction next to each other at the speed of light, relative to each other neither is traveling but you put them heading opposite directions and they are now traveling at twice the speed of light... relative to each other.

Back to the wormhole on the Earth/ship reference. Again, if I am understanding what you are saying, when someone looks into either side of the wormhole, either from the ship or from Earth, everything appears slowed down. Again, that is if I am understanding what you are saying.

But, there is no FTL travel, or even any travel beyond what travel could be hand when looking from one room into another through a door frame. But, that is only if what I understand of wormholes to be correct and they are (put simply) just two sides of a 2 dimensional surface.


I have an idea, instead of replying back to this whole post, lets start from fresh.

A wormhole, is it just two sides of a two dimensional space where traveling between the two puts you from one point to the other by just taking a step through the opening? Where there is actually no space between the sides so no travel is had from entering to exiting. If there is a distance between the two sides, then how far is it? Does the distance between the containers of the openings affect the distance had between the entrance and exit?

Vekseid

Quote from: AriDarrow on June 16, 2012, 04:01:54 PM
Not sure I understood everything you were talking about there, again, using the terminology that I am not fully educated in is only adding to the confusion.

I'm trying to avoid being too snarky, but there is plenty of information publicly available on the Internet to help you familiarize yourself with basic concepts of relativity and, most importantly for our discussion, the concept of frames of reference and the fact that there is no preferred frame.

Quote
I understand that anything going over a distance is travel, but what distance is traveled between the wormhole sides? It's a 2 dimensional space, right?

No, I'm not sure why you would think this.

Take a balloon, pinch part of it together, poke a whole in the middle and seal the edges of the hole. The entire surface, including the 'wormhole', is two-dimensional. Scale this up a dimension, and the wormhole gains one too. The wormhole still has 'depth'.

However, it isn't remotely necessary for our discussion to be at all concerned with that. An ansible or other instant teleport device or reducing the wormhole to such is not necessary, it's just convenient, because it's where math as in the above is the most accurate.

Quote
Someone walking at say, 2mph going into one side is walking the same 2mph coming out the other and if those ends of the wormhole are at opposite sides of the universe, does that mean that they have said ungodly high speed of some FTL travel, or were they only going at 2mph at all points?

It means they have violated the dominant energy condition. It's not generally appropriate to consider any sort of remotely physical (and physicists will probably poke at me for that, but whatever) faster-than-light travel to have 'a speed'. How fast something moves FTL is going to depend on the frame of reference of whomever is observing it. It is, in fact, possible for many FTL mechanisms to appear as slower than light in some suitably absurd frame of reference. Star Trek is the most typical example.

Quote
It's well known that there are galaxies out there that are traveling away from us at speeds greater than the speed of light. It's not due to said galaxy actually traveling faster than the speed of light,

Correct.

Quote
but due to our mutual speeds traveling away from a random set neutral point between our two galaxies and the expansion of the universe.

The former is meaningless. The latter is correct.

Quote
At no point is any going FTL,

Correct.

Quote
but combine all the factors and FTL is observed.

Incorrect. The expansion of spacetime across the Universe creates a horizon, past which we cannot observe. Therefore, while we are well aware of this phenomenon, the actual observation is not possible - the FTL is created because of the deformation of spacetime itself, and galaxies past this horizon cannot send a photon to reach us, thus preventing our observation.

Quote
I don't like saying it this way, but it's one of the better ways of putting, all speed is relative. Two ships traveling in the same direction next to each other at the speed of light, relative to each other neither is traveling but you put them heading opposite directions and they are now traveling at twice the speed of light... relative to each other.

Incorrect. A ship moving at the speed of light would presumably have an infinite horizon, but if you have two ships at rest, and each moves in opposite directions away from each other at .9999 of c, they will each observe the other ship receding from them at a rate defined by the relativistic velocity addition formula.

(.9999 + .9999) / (1 + .9999^2) = 0.999999995

In order for the relative speed between them to actually exceed the speed of light, the expansion of space between them needs to be what pushes them over the edge. Much easier to just drop one into a black hole, but the finality of that might disturb the occupants of said ship.

Quote
Back to the wormhole on the Earth/ship reference. Again, if I am understanding what you are saying, when someone looks into either side of the wormhole, either from the ship or from Earth, everything appears slowed down. Again, that is if I am understanding what you are saying.

While the faces of the wormhole have a relativistic speed difference, yes.

Quote
But, there is no FTL travel, or even any travel beyond what travel could be hand when looking from one room into another through a door frame. But, that is only if what I understand of wormholes to be correct and they are (put simply) just two sides of a 2 dimensional surface.

We could pretend such a wormhole existed as a thought experiment, but it wouldn't change the reality that travel occurs.

Quote
I have an idea, instead of replying back to this whole post, lets start from fresh.

A wormhole, is it just two sides of a two dimensional space where traveling between the two puts you from one point to the other by just taking a step through the opening? Where there is actually no space between the sides so no travel is had from entering to exiting. If there is a distance between the two sides, then how far is it? Does the distance between the containers of the openings affect the distance had between the entrance and exit?

I think you're confused.

You're declaring that it isn't travel by some sort of authorial fiat, like declaring pi to be 4.5 or the Sun to be a giant orange. Mass-energy has vanished from one part of the universe and appeared 100 light-years away. The distance is certainly much shorter in a very localized context, but a distant observer is still going to see a hundred light-years of near-instantaneous travel, and is going to have difficulty ordering the events on either side of the wormhole. That travel - regardless of the mechanism - still occurs.

AriDarrow

Quote from: Vekseid on June 16, 2012, 05:29:22 PM
I'm trying to avoid being too snarky, but there is plenty of information publicly available on the Internet to help you familiarize yourself with basic concepts of relativity and, most importantly for our discussion, the concept of frames of reference and the fact that there is no preferred frame.

At least you said you tried to avoid being snarky. And I could have sworn that I said
QuoteAgain, I've read a few books on this (they made my brain implode) and have watched many videos (Ted.com is awesome) but I've not found any that have really said anything that the way I'm seeing things is backwards.
And that my initial post was made at an early time, so it is very likely that my thinking was clouded.

QuoteNo, I'm not sure why you would think this.

Did a quick check and found a wormhole described in this way. Take a piece of paper and curve it, poke a hole through opposite ends that overlap and that is a simplified example of what a wormhole is, a shortcut between two otherwise very distant points. It may have been incorrect of me saying it was 2 dimensional, so lets say that the distance traveled in the wormhole that we do not have an method today to measure such a small space. I'm not finding any reason why there needs to be space between those two points inside the wormhole, so I simply was saying that it was a 2 dimensional "window" between two points.

QuoteTake a balloon, pinch part of it together, poke a whole in the middle and seal the edges of the hole. The entire surface, including the 'wormhole', is two-dimensional. Scale this up a dimension, and the wormhole gains one too. The wormhole still has 'depth'.

Here it seems like you just contradicted yourself... you say that it's 2 dimensional yet then say it has depth. If we are referring to the entrance and exit to the wormhole having height and width, and it being a 2 dimensional surface, then how could it also have depth?

QuoteIt means they have violated the dominant energy condition. It's not generally appropriate to consider any sort of remotely physical (and physicists will probably poke at me for that, but whatever) faster-than-light travel to have 'a speed'. How fast something moves FTL is going to depend on the frame of reference of whomever is observing it. It is, in fact, possible for many FTL mechanisms to appear as slower than light in some suitably absurd frame of reference. Star Trek is the most typical example.


QuoteThe former is meaningless. The latter is correct.

Again, you confuse me. First you seem to say what I describe, then say it's meaningless then say I am correct again? In the example I was providing, I was saying how two ships moving away from a mutual neutral frame of reference. You say that is meaningless, but it is key to have a frame of reference or another point to compare to, to have any sort of speed. If there was a ship in a total void, how could you tell how fast it was going without anything to reference it to.

My phrasing may have confused you on what I was saying, so lets try this. You are the neutral point. You look left and see a point traveling away from you at 75% the speed of light, you look right and see another point traveling away also at 75% the speed of light. Using you as the neutral point, those two points are traveling away from each other at 150% the speed of light. Those two points being along the same line and opposite directions from each other.

QuoteIncorrect. The expansion of spacetime across the Universe creates a horizon, past which we cannot observe. Therefore, while we are well aware of this phenomenon, the actual observation is not possible - the FTL is created because of the deformation of spacetime itself, and galaxies past this horizon cannot send a photon to reach us, thus preventing our observation.

And I was wrong with using "observe" at that point, but again it seems you are saying exactly what I've been trying to describe.

QuoteIncorrect. A ship moving at the speed of light would presumably have an infinite horizon, but if you have two ships at rest, and each moves in opposite directions away from each other at .9999 of c, they will each observe the other ship receding from them at a rate defined by the relativistic velocity addition formula.

(.9999 + .9999) / (1 + .9999^2) = 0.999999995

In order for the relative speed between them to actually exceed the speed of light, the expansion of space between them needs to be what pushes them over the edge. Much easier to just drop one into a black hole, but the finality of that might disturb the occupants of said ship.

Here again you confuse me. You just said that the expansion of the universe, which I have no problems with and feel I've tried to get that point across, would make their speeds away from each other faster than the speed of light without either actually exceeding that speed itself. But here you say that they wouldn't be traveling away from each other faster than light?

QuoteI think you're confused.

You're declaring that it isn't travel by some sort of authorial fiat, like declaring pi to be 4.5 or the Sun to be a giant orange. Mass-energy has vanished from one part of the universe and appeared 100 light-years away. The distance is certainly much shorter in a very localized context, but a distant observer is still going to see a hundred light-years of near-instantaneous travel, and is going to have difficulty ordering the events on either side of the wormhole. That travel - regardless of the mechanism - still occurs.

And yet I keep thinking you are the one being confusing. In this instance, I wasn't declaring anything, I was asking questions and giving a possible example to try and help clarify the question.

Now, from what I am understanding of what you are trying to tell me, is that it doesn't matter if someone is looking through the wormhole or looking from ship to Earth or back, the same would be viewed?

Wormholes and Time Travel - Ian Morison

That seems contrary to what he describes. Or, in my head it does at least. For what Ian Morison describes, what would be seen through the wormhole would be the same as looking through a window or into the next cabin on the ship. If time dilation was viewed through the wormhole, then the times differences of the two sides of the wormhole would be even greater by the time he returned to Earth.

Honestly, I feel we are saying the same thing, but putting it in such a way as to be confusing to the other. My original question, that being what would be observed, has been answered. I may not like the result, that being time travel is actually possible, but it's a result that I will have to be content with. At least until such a wormhole experiment is possible and tested to see if it really holds true. Thank you for you time.

Vekseid

Quote from: AriDarrow on June 16, 2012, 08:29:07 PM
At least you said you tried to avoid being snarky. And I could have sworn that I said 
And that my initial post was made at an early time, so it is very likely that my thinking was clouded.

Ted is cool but it's not a source of education about stuff like this.

I don't have the time to provide a full education. You're asking me to actually 'argue' very basic relativity with you. You make statements like:

Quote
My phrasing may have confused you on what I was saying, so lets try this. You are the neutral point. You look left and see a point traveling away from you at 75% the speed of light, you look right and see another point traveling away also at 75% the speed of light. Using you as the neutral point, those two points are traveling away from each other at 150% the speed of light. Those two points being along the same line and opposite directions from each other.

This is false. If two things are moving away from each other faster than the speed of light, that means a photon cannot travel to one and then travel to the other. However, a photon most certainly can bounce between these spaceships nigh indefinitely in Minkowski spacetime.

The speed between the ships is given by the relativistic velocity addition equation that I linked. They are moving away from each other at
(.75 + .75) / (1 + .75^2) = .96 of c



Quote
Did a quick check and found a wormhole described in this way. Take a piece of paper and curve it, poke a hole through opposite ends that overlap and that is a simplified example of what a wormhole is, a shortcut between two otherwise very distant points. It may have been incorrect of me saying it was 2 dimensional, so lets say that the distance traveled in the wormhole that we do not have an method today to measure such a small space. I'm not finding any reason why there needs to be space between those two points inside the wormhole, so I simply was saying that it was a 2 dimensional "window" between two points.

Here it seems like you just contradicted yourself... you say that it's 2 dimensional yet then say it has depth. If we are referring to the entrance and exit to the wormhole having height and width, and it being a 2 dimensional surface, then how could it also have depth?

Those are two-dimensional wormholes, in two-dimensional universes. They both have some concept of depth in those universes. You add a dimension to the universe, you add a dimension to the wormhole along with it.

It's not terribly relevant, regardless.

Quote
Again, you confuse me. First you seem to say what I describe, then say it's meaningless then say I am correct again? In the example I was providing, I was saying how two ships moving away from a mutual neutral frame of reference. You say that is meaningless, but it is key to have a frame of reference or another point to compare to, to have any sort of speed. If there was a ship in a total void, how could you tell how fast it was going without anything to reference it to.

There does not need to be a mutual neutral frame of reference for the discussion - it merely adds complexity to an issue that, plainly, doesn't need to have complexity added. It's irrelevant, unless you're picking a point which is within both points' event horizons. But such points are merely useful for discussion about the issue - they aren't necessary to discuss spacetime's expansion and its effects.

Quote
My phrasing may have confused you on what I was saying, so lets try this. You are the neutral point. You look left and see a point traveling away from you at 75% the speed of light, you look right and see another point traveling away also at 75% the speed of light. Using you as the neutral point, those two points are traveling away from each other at 150% the speed of light. Those two points being along the same line and opposite directions from each other.

See above. They are not moving apart at 1.5c, they are moving apart at .96c.

Quote
And I was wrong with using "observe" at that point, but again it seems you are saying exactly what I've been trying to describe.

You're still having issues with normal motion versus recession caused by the expansion of spacetime.

Quote
Here again you confuse me. You just said that the expansion of the universe, which I have no problems with and feel I've tried to get that point across, would make their speeds away from each other faster than the speed of light without either actually exceeding that speed itself. But here you say that they wouldn't be traveling away from each other faster than light?

They would not. If two things are moving apart faster than light, that means a photon cannot cross the gap between them. It requires an exceptional situation for this to occur.

Main subject to look up, here, is Lorentz Contraction. You move at .86c in a direction, distance is compressed along that direction by a factor of 2.  This includes behind you. Thus, you need a different equation when adding the velocities, as I linked.

Quote
And yet I keep thinking you are the one being confusing. In this instance, I wasn't declaring anything, I was asking questions and giving a possible example to try and help clarify the question.

Now, from what I am understanding of what you are trying to tell me, is that it doesn't matter if someone is looking through the wormhole or looking from ship to Earth or back, the same would be viewed?

Not sure what you mean by 'same' here.

Quote
Wormholes and Time Travel - Ian Morison

That seems contrary to what he describes. Or, in my head it does at least. For what Ian Morison describes, what would be seen through the wormhole would be the same as looking through a window or into the next cabin on the ship. If time dilation was viewed through the wormhole, then the times differences of the two sides of the wormhole would be even greater by the time he returned to Earth.

Having it return to Earth makes for fewer headaches, like trying to seriously answer 'could his wife actually look at Andromeda after only four hours?'

Regardless, it's true - 4 hours later, 2.5myear difference, 8 hours, 5myear.

Quote
Honestly, I feel we are saying the same thing, but putting it in such a way as to be confusing to the other. My original question, that being what would be observed, has been answered. I may not like the result, that being time travel is actually possible, but it's a result that I will have to be content with. At least until such a wormhole experiment is possible and tested to see if it really holds true. Thank you for you time.

And that isn't, actually, what I said.

AriDarrow

Quote from: Vekseid on June 16, 2012, 10:15:06 PM
And that isn't, actually, what I said.

Ok, will try one more time, starting over from scratch. We know the chorus by now, ship traveling at near speed of light, wormhole on ship and on Earth.

Now, to put my original question as simply as I can, what would a person see when looking into the wormhole on the ship?

My challenge to you, answer it as if a 12 year old asked you that question.

I put it this way due to a lot of frustration I am having at your answers seeming to contradict other answers you give also in you saying I am wrong when I try and put what you say into a more simple form that seems nearly identical to what you say, but again say I am wrong. I would really like a 3rd person to read over this whole line and say if I am just seeing what isn't there or if some of your answers really do seem to contradict each other. Regardless, this will most likely be one of, if not my last, post along this thread, but I do wait with baited breath for your final attempt at the challenge I have put forth.

AndyZ

I've been watching this thread, and here's a question from me:

Quote from: Vekseid on June 16, 2012, 05:29:22 PM
Incorrect. A ship moving at the speed of light would presumably have an infinite horizon, but if you have two ships at rest, and each moves in opposite directions away from each other at .9999 of c, they will each observe the other ship receding from them at a rate defined by the relativistic velocity addition formula.

(.9999 + .9999) / (1 + .9999^2) = 0.999999995

In order for the relative speed between them to actually exceed the speed of light, the expansion of space between them needs to be what pushes them over the edge. Much easier to just drop one into a black hole, but the finality of that might disturb the occupants of said ship.

Please point out my flaw.

Now, let's say that at point O, two spaceships blast off away from each other, each going at .9999 C.  To an observer watching at point O, the distance between the two would increase at a rate equal to 1.9998 C.  This would mean that as O watches (ignoring the fact that he needs light to watch and that he would have to use redshifting and calculations to figure out actual time), the light created by the rocket of one ship would never be able to bounce off of the other.  The two ships would not be able to see each other.

Now, from the perspective of one ship, the distance between the two is increasing at a rate of  0.999999995 C.  This would mean that, however long it would take, light created by the rocket of one ship would be able to bounce off of the other ship.  The people on each ship would still be able to see each other.

Please point out my flaw.
It's all good, and it's all in fun.  Now get in the pit and try to love someone.

Ons/Offs   -  My schedule and A/As   -    My Avatars

If I've owed you a post for at least a week, poke me.

Vekseid

Quote from: AndyZ on June 17, 2012, 02:21:03 AM
I've been watching this thread, and here's a question from me:

Please point out my flaw.

Now, let's say that at point O, two spaceships blast off away from each other, each going at .9999 C.  To an observer watching at point O, the distance between the two would increase at a rate equal to 1.9998 C.  This would mean that as O watches (ignoring the fact that he needs light to watch and that he would have to use redshifting and calculations to figure out actual time), the light created by the rocket of one ship would never be able to bounce off of the other.  The two ships would not be able to see each other.

Now, from the perspective of one ship, the distance between the two is increasing at a rate of  0.999999995 C.  This would mean that, however long it would take, light created by the rocket of one ship would be able to bounce off of the other ship.  The people on each ship would still be able to see each other.

Please point out my flaw.

Do a mental experiment for a moment.

Give person O a laser pointer, and a mirror on the back of each ship (call them A and B)

O points the laser at A's mirror. It eventually reaches A (because we all agree that it can reach it. Hits it, and bounces back - extremely redshifted, but it's still bouncing back. Because it's returning at the speed of light from all frames of reference, it will eventually reach O's position again, and assuming O steps out of the way, will be racing towards B - still at the speed of light. Possibly redshifted beyond any hope of retrieval, but maybe it's an X-ray laser or something. The light bounces off of B's mirror, and O will see it, some significant time later. Or O can step out of the way and watch it hit A again, let it bounce back to B again... as often as O likes until it's so redshifted that it isn't able to be distinguished from background radiation.

Does that help?

AndyZ

It's all good, and it's all in fun.  Now get in the pit and try to love someone.

Ons/Offs   -  My schedule and A/As   -    My Avatars

If I've owed you a post for at least a week, poke me.

Vekseid

There are a lot of fun paradoxes with Relativity.

The Ladder Paradox:

A farmer has a barn, 20 units long. A runner has a ladder, 30 units long.

The runner starts running towards the open barn at .86 of c. The barn appears, to the runner, to be 10 units long.

To the farmer, the runner's ladder is 15 units long. Once the runner is entirely inside, the farmer has had enough of these relativistic hijinx and closes the barn doors, and the runner is completely inside the barn - 15 unit ladder and all.

The farmer's son considers the runner a friend, or at least doesn't feel like cleaning up the mess. He presses the button, the doors open an instant later, and the runner continues on his way.

How is this possible?

The answer is of course on-line, but there's at least one clue to it in this thread.

Quote from: AriDarrow on June 17, 2012, 01:54:59 AM
*snip* I do wait with baited breath for your final attempt at the challenge I have put forth.

If all I'm going to get are sarcastic quips, I'll spend my time on others, instead.

AriDarrow

Quote from: AndyZ on June 17, 2012, 02:21:03 AM
I've been watching this thread, and here's a question from me:

Please point out my flaw.

Now, let's say that at point O, two spaceships blast off away from each other, each going at .9999 C.  To an observer watching at point O, the distance between the two would increase at a rate equal to 1.9998 C.  This would mean that as O watches (ignoring the fact that he needs light to watch and that he would have to use redshifting and calculations to figure out actual time), the light created by the rocket of one ship would never be able to bounce off of the other.  The two ships would not be able to see each other.

Now, from the perspective of one ship, the distance between the two is increasing at a rate of  0.999999995 C.  This would mean that, however long it would take, light created by the rocket of one ship would be able to bounce off of the other ship.  The people on each ship would still be able to see each other.

Please point out my flaw.

But, with the expansion of space, wouldn't the combination of the distance between the two ships as well as the rate at which the space between them expanding make it, at some point, impossible for the ships to see each other?

piece

QuoteThere are a lot of fun paradoxes with Relativity.

The Ladder Paradox:

A farmer has a barn, 20 units long. A runner has a ladder, 30 units long.

The runner starts running towards the open barn at .86 of c. The barn appears, to the runner, to be 10 units long.

To the farmer, the runner's ladder is 15 units long. Once the runner is entirely inside, the farmer has had enough of these relativistic hijinx and closes the barn doors, and the runner is completely inside the barn - 15 unit ladder and all.

The farmer's son considers the runner a friend, or at least doesn't feel like cleaning up the mess. He presses the button, the doors open an instant later, and the runner continues on his way.

How is this possible?

Wouldn't this go back to the concept of that frame of reference, and with the original question Ari posed about what would be seen via the wormhole?

I am by no means highly educated in this subject, but I'd like to take a stab here. And I'm perfectly willing to sound foolish if I learn from it! I have heard of this paradox before, but I don't fully grasp all the mechanics and principles behind it.

So, here goes. The ladder appears to be moving really, really fast to the barn, and the barn would appear to be moving really, really fast to the ladder. So the ladder, as seen from the barn, would be 15 units long, which would fit in the actual 20 unit space of the barn. From the barn's frame of reference, the ladder fits.

But what happens from the ladder's frame of reference? That's what I'm not completely sure how to reconcile, because going from the ladder's point of reference, the barn would only be 10 units in length. Help, please?

Vekseid

The answer has been hinted at in this thread.

Blatant hint

'Now' is relative. Events that are simultaneous from one perspective are not necessarily simultaneous from another, and spacelike events (events not within each others' light cones) cannot have any special ordering according to relativity - A->B or B->A depends on the observer.

Constructing a mechanism that permits FTL and maintains Causality requires that there be a correct special ordering. However, people would still observe the incorrect ordering from some frames.

piece

Oh! Now it makes much more sense to me! The paradox isn't just in the ladder fitting in the barn, but also the boy opening the door.

Vekseid

Not exactly.

Spoiler: Click to Show/Hide

The doors closing and opening are separate events, A and B. They are not within each others light cone, so there is no true ordering as to which is actually first. To the farmer and his son, events A and B are simultaneous. Now thing about what the runner will see.
[/quote]

piece

Spoiler: Click to Show/Hide
So...

If A and B are simultaneous from the perspective of the farmer and his son, in the barn, they might not be from the runner's perspective with the ladder? If there's no ordering of A and B from the runner's perspective, the two events can happen in a different order than what the farmer saw.

Which would mean B can precede A from a different frame of reference.

Without an understanding of the math behind it, I'm still looking at this from a weak position, but I think I see how that would eliminate the paradox of the ladder not fitting for the runner. It wouldn't ever actually be closed in, if the rear doors opened before the front door closed, if I'm understanding rightly?

Vekseid

Spoiler: Click to Show/Hide

Easier to call them far and near rather than rear and front.

But yes, you have it. : ) The far door closes and opens before the near door does, from the runner's perspective. A runner running at such speeds in the opposite direction would claim the opposite ordering.

Kate

#51
The most awkward thing about physics is just one thing

Until a proven "theory of everything" exists , its hard to know with certainty ANYTHING.

Current theories of quantum mechanics and Relativity are very good STATISTICALLY, ie "these rules seem to explain ALOT, reality "seems" like this. Strangely subjective and objective reality become very different when some things get extreme

(extremely small, extremely fast, extremely dense, extremely energetic, extremely cold)
The more of an effort we use to pin down specifics the more stranger weird things seem to be all the "hard end points for any theory, are primarily more question marks than anything else"

Even if a "theory of everything" did exist, there may be SOME things that are unknowable practically.

Physics and science generally are VERY refined, educated guesses, many breakthroughs happen when previously existed notions that are accepted scientifically are questions and re-examined (or dismissed as "not the same context" due to a particular difference that wasn't deemed relent or considered before hand).

One large one is "universal constants", where did they come from ? Are they really "constant?" are they just a platua of values of a plane in a higher relativity that changes ( it just "seems flat" from where we are ). There was a recent article of some area of space where it seemed the universal constant for something was SLIGHTLY different to other places. What stunned the scientific community is that it was a basic "fundamental universal constant", that they thought all the universe was sharing.

"what is common" concerning laws of physics may vary from PLACE, TIME, and strangely ... the perspective that's experiencing it.

AndyZ

Earth is spinning at roughly 1,000 miles an hour, revolving around the Sun at some speed (which probably varies to some degree), and the Sun moves around the galaxy as well.  All this adds up to some value which is greater than zero at which we're constantly moving, when compared to an absolutely fixed point in space.

I know that according to the special theory of relativity, the faster you speed up, the more time seems to speed up as well, so that you're aging less.  Theoretically, the converse should be true if you slow down, so that time would seem to slow down and you'd age more, right?

Has anyone ever tried to test it from this angle?  Reaching c would be impossible, but it should be much easier to get to a speed of absolute zero, assuming that from the perspective of a fixed point in space, we're travelling at less than half c.
It's all good, and it's all in fun.  Now get in the pit and try to love someone.

Ons/Offs   -  My schedule and A/As   -    My Avatars

If I've owed you a post for at least a week, poke me.

Vekseid

Quote from: Kate on June 28, 2012, 10:56:59 PM
The most awkward thing about physics is just one thing

*snip*

Please see this thread, thank you.

Quote from: AndyZ on June 28, 2012, 11:53:52 PM
Earth is spinning at roughly 1,000 miles an hour, revolving around the Sun at some speed (which probably varies to some degree), and the Sun moves around the galaxy as well.  All this adds up to some value which is greater than zero at which we're constantly moving, when compared to an absolutely fixed point in space.

It's really hard to know whether or not there even is such a thing, but we're moving at ~600 kilometers/second through the Intergalactic Medium.

Quote
I know that according to the special theory of relativity, the faster you speed up, the more time seems to speed up as well, so that you're aging less.  Theoretically, the converse should be true if you slow down, so that time would seem to slow down and you'd age more, right?

-You- wouldn't notice the change. People observing you, however, would. And you would observe the same of them until one of you decided to take a trip back and bring your concept of 'now' into agreement. Whoever undergoes the acceleration and deceleration to meet up will be the one who ages less.

Quote
Has anyone ever tried to test it from this angle?  Reaching c would be impossible, but it should be much easier to get to a speed of absolute zero, assuming that from the perspective of a fixed point in space, we're travelling at less than half c.

It's not necessary to find any such 'fixed point' in space (Relativity explicitly denies that such a thing, were it to exist, has any relevance). 600 km/sec in any direction would do. But we have much finer ways to measure such things. GPS needs to make relativistic adjustments as-is. A more startling example is decay particles from reactions in the upper atmosphere that, were Relativity wrong about this, shouldn't even reach the ground to be detected. And yet there they are.

More mundanely, just find a magnet. Or a piece of gold. Or some mercury.

Magnets are the result of electric charges moving at relativistic speeds.

Gold's color and mercury being a liquid is the result of their inner electron shells being contracted due to moving at high relativistic velocities (if you're wondering the obvious question there, an electromagnetic 'black hole' is created at about 174 protons, IIRC. Such an atom could not have electrons in its lowest orbital at rest, it'd eat them and start losing charge, probably violently >_>).

Oniya

The thing is, velocity is a vector:  it has direction and magnitude.  The magnitude is commonly referred to as 'speed', and must always be positive. 

Time dilation is a factor which changes based on the speed difference between two inertial frames of reference.  Two frames that are stationary, relative to each other (two cars going the same direction at the same speed, for example), have no time dilation - it's a factor of 1.  As one gets faster than the other, each perceives the other one's clock as moving slower, because if A assumes that it is the stationary one, then B is moving at the increased speed in one direction or another.  If B assumes it is the stationary one, A is moving at the increased speed.

In short, the best you can do by slowing down is to break even.

"Language was invented for one reason, boys - to woo women.~*~*~Don't think it's all been done before
And in that endeavor, laziness will not do." ~*~*~*~*~*~*~*~*~*~*~Don't think we're never gonna win this war
Robin Williams-Dead Poets Society ~*~*~*~*~*~*~*~*~*~*~*~*~*~Don't think your world's gonna fall apart
I do have a cause, though.  It's obscenity.  I'm for it.  - Tom Lehrer~*~All you need is your beautiful heart
O/O's Updated 5/11/21 - A/A's - Current Status! - Writing a novel - all draws for Fool of Fire up!
Requests updated March 17

AndyZ

Oh, so it's just perception of slowing down, and there's no actual time dilation?  That would make sense.  Otherwise one would have to go slower or faster than the other in time, based on which was moving faster in velocity.
It's all good, and it's all in fun.  Now get in the pit and try to love someone.

Ons/Offs   -  My schedule and A/As   -    My Avatars

If I've owed you a post for at least a week, poke me.

Vekseid

Quote from: AndyZ on June 29, 2012, 12:43:42 AM
Oh, so it's just perception of slowing down, and there's no actual time dilation?  That would make sense.  Otherwise one would have to go slower or faster than the other in time, based on which was moving faster in velocity.

Time dilation is something that occurs between different frames of reference. It does not occur in your frame of reference.

And they both are going slower, until one decides to (and expends the energy to) meet up.

Kate

#57
Laymen view (feel free to correct me):

Time "slows down" for the object that is fast OR near a very heavy object to the "else". (if it was faster one would age faster)

What is hard to get is that if EVERYTHING is relative... and you speed yourself up relative to the world say 0.9999 the speed of light, you age SLOWER compared to the world, when you return to the world more time has passed for THEM.

Your younger than your twin brother who stayed on the world.

BUT the world was moving 0.9999 the speed of light compared to YOU.

So why doesn't the world age slower (ie those on it younger than you) to the Astronaut that returned as it was "moving at 0.9999" the speed of light for a period of X compared to the star-ship traveler ?

Is it due to "relative speed to the center of the universe", ie the one"reference point" that is universal?

Now if everything is rotating, and you go in the opposite direction to everything else, compared to the center of the universe your at te same speed. But you may be traveling at 0.5 c or something (compared to everything else around you) then what ?

It has to do with the "arrow of time", each quantum thing has its own "arrow of time" the less entangled you are with other things the more ind-pendant that arrow is (ie quantum things happen instantly going forward and backwards in time) that's when they are are only entangled with each other and nothing ELSE. What your entangled with you SHARE an arrow of time with.

But the larger you are the more things your entangled with, that why quantum fluctuations are smoothed on mass. (ie people dont generally experience quantum phenomena) - arrows of time is "resolved" ie shared, like a super-fluid / bosen (spelling ?) concentrate.

How to picture this ? a large sheet of iron, normally parts of the iron can have its magnetic field all over the place at random.
This most pieces of iron are NOT magnets.

But if you expose it to a LARGE magnetic field / surge of electicity ... over time all of its components like up and "share" a "magnetic arrow". And in turn becomes the SOURCE of the magnetic field that "keeps the arrow pointing this way"

Now swap the "magnetic arrows (field)" with "time arrow"

If all you knew was the sheet when "magnetised in this state" the arrow of time would seem "constant and universal and apply everywhere to everyone" from what you "SEE" that is true, everything else interacting with you is "sharing that background time arrow", but we dont "see everything" 90 percent of the unverse we feel with gravity but cant detect in other ways.

"the else" may not share enough constants to be "entangled" enough with us to even notice it in ways other than gravity. Ie two large sheets of iron had VERY different arrows (or whatever), interacting with each other may be HARD (ie effectively a different "dimension"

So in a localized area of space where everything can see each other "the arrow of time is shared", areas "not so entangled" may have the arrow drift differently.

Now if its "not so entangled" it interacts LESS with other things. Typically when things are heated / more energy they react MORE with other things, so the faster you go the more likely it is your hitting stuff etc (chemical reactions 1 degree higher are reacting 10 times as quickly, 2 degrees 100 times, three degrees Celsius 1000 times as fast a chemical reaction change compared to something 3 degrees cooler. ). This flow of thought is a "catch" though to this issue, and where rules of thumb / general knowledge / common sense / assuming shared contexts doesn' serve you.

Something moving at X mps relative to "BACKGROUND time arrow (a time-magnetic field) while entangled with it"
experiences what we know as "relativity". "Disentangling that" or having a change of "background arrow"

=> One of MANY hopes that a "theory of everything" (Assuming its knowable from what we can experience or do) .. may provide.

Now if everything that exists is vibrating in 11 dimensions (current view shared by most, not "the truth"). 3 or so are Drawn out and large ( spacial ) the others are very tightly bunched together. How many degrees of freedom things have due to these dimensions is amazing, now these "dimensional vibrations" behave and what they can do, how they change things is something that hopefully the theory of everything can provide.

In short, until we have a "workable" theory of everything "seems" is as close as we can get. If we do find a "theory of everything" it may just apply to our CURRENT conditions, and not explain that much of the "else" as we hoped, nor be able to predict when the current conditions will change and invalidate our current "theory of everything".

Everything gets stranger and stranger the more you know, the less you realize you know.

One famous quote that makes me believe we may be able to "get it" anyway was famous scientist who said something like

"The strangest thing I feel about the universe is that it seems to be bending over backwards TRYING to show its secrets to us"

For any "firm and hard theory" there seem to be contradictions. Like English "i before e except after c .. with the exceptions of this word oh and that one .. oh and this one also ... and also this one, oh wait another" ... if so many exceptions exist when do rules become useful at all ? For equations in physics Trying to have them minimalistic and still inclusive of describing everything is

... "When do you apply X assumption, rule Y ?"

... its a bitch and has our physicists in a state where they don't all agree on ANYTHING.

This MAY be true for different parts of the universe, or very differnt perspectives (Real in their own regard)
they may not agree on ANYTHING.

AndyZ

Quote from: Vekseid on June 29, 2012, 12:51:19 AM
Time dilation is something that occurs between different frames of reference. It does not occur in your frame of reference.

And they both are going slower, until one decides to (and expends the energy to) meet up.

Yeah, that explains my confusion.  There was some Sci Fi story (I think part of the Ender's Game series) where they would travel at close to the speed of light in order to slow down, so somebody stayed young even though hundreds of years passed on Earth.  Knowing that it's an equal and opposite perception and that time isn't actually affected helps make things make a lot more sense.
It's all good, and it's all in fun.  Now get in the pit and try to love someone.

Ons/Offs   -  My schedule and A/As   -    My Avatars

If I've owed you a post for at least a week, poke me.

Vekseid

Quote from: AndyZ on June 29, 2012, 12:55:25 AM
Yeah, that explains my confusion.  There was some Sci Fi story (I think part of the Ender's Game series) where they would travel at close to the speed of light in order to slow down, so somebody stayed young even though hundreds of years passed on Earth.  Knowing that it's an equal and opposite perception and that time isn't actually affected helps make things make a lot more sense.

One way to think of this is, several hours later, they're a hundred light years away, but only a few seconds have passed on Earth. Earth starts chasing them at a similarly obscene velocity a few of Earth's hours later. Earth will catch up to them in several hours from Earth's frame - but will take centuries from the perspective of the people in the ship for Earth to reach them.

Naturally, at that point they'll have traveled a ridiculous distance from Earth's original frame of reference.

Quote from: Kate on June 29, 2012, 12:51:30 AM
So why doesn't the world age slower (ie those on it younger than you) to the Astronaut that returned as it was "moving at 0.9999" the speed of light for a period of X compared to the star-ship traveler ?

The one who undergoes the actual acceleration is the one that experiences the time dilation, generally. The Astronaut accelerated to .9999c, decelerated to 0, than accelerated back.

Quote
Is it due to "relative speed to the center of the universe", ie the one"reference point" that is universal?

There is no such point. There can be no such point, as parts of the Universe are receding from us faster than light. You can make the math work by declaring the IGM to be a 'special frame', but that space is still expanding, and for our purposes, it isn't necessary to consider. That 'special frame' is no longer a single point, it's an abstraction of the Universe in its entirety.

Quote
Now if everything is rotating, and you go in the opposite direction to everything else, compared to the center of the universe your at te same speed. But you may be traveling at 0.5 c or something (compared to everything else around you) then what ?

Again, there is no such thing as the Universe having a 'center', except for one that is omnipresent.

Quote
It has to do with the "arrow of time", each quantum thing has its own "arrow of time" the less entangled you are with other things the more ind-pendant that arrow is (ie quantum things happen instantly going forward and backwards in time) that's when they are are only entangled with each other and nothing ELSE. What your entangled with you SHARE an arrow of time with.

The 'arrow of time' merely defines our perception of time, in that entropy represents an irreversible process and thus we can conceive of a 'past' and a 'future'. It is not, in and of itself, time.

Quote
*snip*

I think you've been reading stuff that has confused more than enlightened you. : /

Quote
This MAY be true for different parts of the universe, or very differnt perspectives (Real in their own regard)
they may not agree on ANYTHING.

All frames of reference, after working the proper math, will agree on what is happening to all other frames of reference, at least within the observable Universe to some suitably obscene precision. While e.g. a quasar moving away from us at .99 of c seems to be moving forward in time rather slowly, we know that a sapient observer living 'now' in said quasar will see the Milky Way much the same way (if our rather uneventful galaxy can be noticed so easily from such a distance, anyway - just pretend they have a better funded space program with bigger telescopes >_>).

While we do permit the idea that e.g. the 'Universal constants' may be able to 'change', e.g. the fine structure constant might have changed on the order of something like one part per billion or less over aeons - it's of somewhat limited relevance to this discussion. For one, we can observe that and take it into account, if it exists, but for two, even on the observable large-scale structure of the Universe, these variations, if they exist, are extremely tiny.

Kate


AndyZ, one thing i thoroughly agree with V on is this post -

this thread

(My subjective stance)

Regardless if V has the "right take" on what happens if the theory of relatively is applied in your examples,
and indeed if relativity does seem to hold true in the macro sense in the same way as it does for smaller objects,
or higher order consciousness also.

you asked in a thread titled "Theory of relativity questions" but its also titled "physics" (Exploration of the real)

Like what V implied in his post, Concerning physics, its NOT the theory of relativity and quantum mechanics, theory of relativity and quantum mechanics are just imperfect (but VERY useful) TOOLs for physics at the current time.

Know that science often has a LIST of exceptions which doesnt seem to make common sense with the currently established understanding of some "Universal rules". "Faith" in any one of them that is thorough isnt met, if it was all would agree to one take of the theory of everything.

Still relativity has been shown many many times to be highly accurate at guessing what happens if ... then giving it a go to confirm - its a very close approximation to a lot of phenomena.

To me Stay inquisitive be open minded, stay awed, what "feels right and true and wonderous" and 'what this space" is something I hope you view as important as the views of any "experts" in a field.

Maths and science fields, all experts have a subjective lean on what they believe also, and yes although their work does remove filtered bias lenses most have, they are not immune to be ignorant or dismissive of the relevance of their own set.

Oniya

Quote from: Kate on June 29, 2012, 09:57:40 PM
(My subjective stance)

Regardless if V has the "right take" on what happens if the theory of relatively is applied in your examples,
and indeed if relativity does seem to hold true in the macro sense in the same way as it does for smaller objects,
or higher order consciousness also.

One thing here - the 'take' that you refer to is not merely Vekseid's, or mine, or any one person's.  This is the 'take' of decades of research.  The general theory has been verified in ordinary gravitational fields. Citation.  Time dilation was observed - get that, observed - by Ives and Stillwell in 1938, and with more precision as better instruments came on the scene Citation (2010).  Labeling these theories as 'imperfect' is disingenuous at best.
"Language was invented for one reason, boys - to woo women.~*~*~Don't think it's all been done before
And in that endeavor, laziness will not do." ~*~*~*~*~*~*~*~*~*~*~Don't think we're never gonna win this war
Robin Williams-Dead Poets Society ~*~*~*~*~*~*~*~*~*~*~*~*~*~Don't think your world's gonna fall apart
I do have a cause, though.  It's obscenity.  I'm for it.  - Tom Lehrer~*~All you need is your beautiful heart
O/O's Updated 5/11/21 - A/A's - Current Status! - Writing a novel - all draws for Fool of Fire up!
Requests updated March 17

Kate

#62
You have a point O,

But wasn't implying he was wrong  :)

WhiteyChan

D'aww. I just missed out on all the fun of this topic. I'm doing an MSc in Physics, and one of my modules is 'Theories of Matter, Space and Time', which covers mostly Special Relativity (applied to motion, electromagnetism, and the standard model) and Lagrangian physics. I could have had fun with this topic. Anyway yes, Veks is pretty much right on everything.